Help!!!!! plz ill award as brainliest

A piggy bank contains $2.65 in nickels and dimes. If the number of nickels is 7 less than three times the number of dimes, how many of each kind of coin are there?

A traveler covered 4/5 of his journey with bicycle and the rest of the distance on foot. If the whole journey was 160 km, how much distance did he cover on foot?

Answers

Answer 1

Answer:

x = 12 dimes and 3x-7 = (3*12)-7 = 29 nickels in the piggy bank

Step-by-step explanation:

Hope this helps

Answer 2

Answer:

see below

Step-by-step explanation:

1.

total = $2.65 = d + n

let d = dimes

   n = nickels

nickels = 3d - 7

plugin values into the formula:

2.65 = d + 3d - 7

2.65 + 7 = 4d

d = 9.65

       4

d = $2.41

nickels = 3d - 7

             = 3(2.41) - 7

          n = $0.24

----------------------------------------------------------

2.

since Cindy ate 30, less than Ann and Ben together so, the equation is

7x = 5x + 5x - 30

7x - 10x = -30

x = 30/3

x = 10

ann   5 x 10 = 50

ben   5 x 10 = 50

cindy 7 x 10 = 70  

total            = 170


Related Questions

How many 1/8 teaspoons are equal to 4 3/4 teaspoons?

Answers

Answer:

24 1/8 teaspoons should be equal to 4 3/4 teaspoons.

Step-by-step explanation:

We let t be the number of 1/8 teaspoons

[tex] \frac{1}{8} t = 4 \times \frac{3}{4} \\ [/tex]

The equation above just means if I have t 1/8 teaspoons is equal to 4 3/4 teaspoons

[tex] \frac{1}{8} t = 4 \times \frac{3}{4} \\ \frac{1}{8} t = 3 \\ t = 24[/tex]

please help me with this

Answers

Answer:

C

Step-by-step explanation:

First, add 10% of $7,250 to itself:

7250 + 725 = 7975

Then, take ten percent of 7975:

7975 / 10 = 797.5

Add that to 7975:

7975 + 797.5 = 8772.5

HEEELLLPPP

Let S be the universal set, where:S = {1, 2, 3, ..., 18, 19, 20}
Let sets A and B be subsets of S, where:

Set A = {4, 6, 8, 15, 16, 17, 18, 19, 20}

Set B = {1, 2, 4, 6, 7, 11, 17, 18}

Find the following:

LIST the elements in the set (AUB):
(AUB) = {

Enter the elements as a list, separated by commas.

LIST the elements in the set (A n B):
(An B) = {

Enter the elements as a list, separated by commas.

Answers

HOPE THIS IS THE ANSWER FOR YOUR QUESTION

is 17/100 greater or lesser than 8.7

Answers

Answer:

I believe its less than

A piece of lumber is 17 feet long. You cut the lumber into logs that are 18 inches long.

How many inches of lumber are remaining?

Answers

17 feet x 12 inches per foot = 204 total inches.

204 inches / 18 inches = 11.33

They cut 11 pieces and had 12x 0.33 = 4 inches remaining.

Answer:

186 inches

step by step: multiple 17x12 then subtract ur  answer from 18 theres ur answer

if i have 789.4 and my mom has 449.7 how much will we both have

Answers

Answer:

1239.1

Step-by-step explanation:

x + y = z

x = 789.4 (your money)

y = 449.7 (Your mom's money)

z = Total amount of money.

789.4 + 449.7 = z

789.4 + 449.7 = 1239.1

z = 1239.1

Assignment Active Simplifying Irrational Zeros of a Quadratic Function What are the zeros of the function f(x) = x^2 + 8x + 4, expressed in simplest radical form?​

Answers

Answer:

(x+2)^2

Because x^2+2*x*4+2^2

whats the largest even 6-digit different number ?​

Answers

Answer:

What does the question even mean?

is it like "What is the largest even 6-digit number?"

If it is so.. then,

999998...

Find 1/6 of an hour?

Answers

Answer:

10 minuites

Step-by-step explanation:

A researcher studying stress is interested in the blood pressure measurements of chief executive officers (CEOs) of major corporations. He believes that the mean systolic blood pressure, μ, of CEOs of major corporations is different from 136 mm Hg, which is the value reported in a possibly outdated journal article. He plans to perform a statistical test. He measures the systolic blood pressures of a random sample of CEOs of major corporations and finds the mean of the sample to be 126 mm Hg and the standard deviation of the sample to be 18 mm Hg.Based on this information, answer the questions below.What are the null hypothesis and alternative to be used for the test (ie, less than, less than or equal to etc)H0 is μ= ____ _______( 18,136, 126) pick oneH1 is μ = _____ _____ (18,136,126) pick oneIn the context of this test what is the type I error? Pick one in the parathensisA type I error is ______(rejecting or failing to reject) that the μ is____________ (less than , equal to, etc) (18,136,126) when in fact μ is__________ ( less than, greater to , etc) (18,136,126)Suppose the researcher decides not to reject the null hypothesis. What sort of error might he be making? (type I or type II)

Answers

Answer:

Null Hypothesis, [tex]H_0[/tex] : [tex]\mu[/tex] = 136 mm Hg  

Alternate Hypothesis, [tex]H_A[/tex] : [tex]\mu\neq[/tex] 136 mm Hg

In this context type I error is rejecting that the μ is equal to 136 mm Hg when in fact μ is equal to 136 mm Hg.

Step-by-step explanation:

We are given that the mean systolic blood pressure, μ, of CEOs of major corporations is different from 136 mm Hg, which is the value reported in a possibly outdated journal article.

He measures the systolic blood pressures of a random sample of CEOs of major corporations and finds the mean of the sample to be 126 mm Hg and the standard deviation of the sample to be 18 mm Hg.

So, Null Hypothesis, [tex]H_0[/tex] : [tex]\mu[/tex] = 136 mm Hg     {means that the mean systolic blood pressure, μ, of CEOs of major corporations is 136 mm Hg}

Alternate Hypothesis, [tex]H_A[/tex] : [tex]\mu\neq[/tex] 136 mm Hg      {means that the mean systolic blood pressure, μ, of CEOs of major corporations is 136 mm Hg}

Type I error states that the null hypothesis is rejected when in fact the null hypothesis was true. So, in this context type I error is rejecting that the μ is equal to 136 mm Hg when in fact μ is equal to 136 mm Hg.

Suppose the researcher decides not to reject the null hypothesis. It means the researcher is making a type I error.

Luke is five years younger than Jason. Two
year ago, the sum of their ages was 33. Howe
old are they?

Answers

Answer:

16 and 21

Step-by-step explanation:

two years ago they were 14 and 19 which we can figure out using the equation (33-5)/2=14 and 33-14=19. so we add 2 to both of those numbers to get their current ages.

Justin wants to buy 100 bottles of juice to sell at a school Basketball game. The juice costs 1.39 per bottle. Justin wants to multiply 1.39$ by 100 to find the total cost

Answers

Answer:

3

Step-by-step explanation:

Carry out the indicated operations and simplify where possible:


[tex]x(8x-8y)^-^1+y(8y-8x)^-^1[/tex]

Answers

Answer:

1/8

Step-by-step explanation:

Solving in steps:

x(8x - 8y)⁻¹ + y(8y - 8x)⁻¹ = x/8(x-y) +y/8*(-(x-y))x/8(x-y) - y/8(x-y) = (x -y)/8(x-y) =1/8

Answer is 1/8

What is an angle?
A corner
Two rays conjoined with a vertex
The way I hold my phone to take my selfies
Two segments
What is an angle

Answers

Answer:

Two rays conjoined with a vertex

In Geometry, an angle is when two rays intersect/conjoin with a vertex. The answer is Two rays conjoined with a vertex. Hope it helps!

Answer:

an angle can be defined as the figure formed by two rays meeting at a common end point.

An angle is represented by the symbol ∠.

Step-by-step explanation:

¡ Mrk as brainliest Pls and follow me

I'm new here......

I need help with this problem

Answers

Answer:correct

Step-by-step explanation:

The answers are D and F

A species of fish was added to a lake. The population size P (t) of the species can be modeled by the following function, where t is the number of years from the time the species was added to the lake

Answers

Answer:

p+t i dont know the question dose not make sence

Step-by-step explanation:

What is 4,708.06 to the nearest thousand ?

Answers

Answer: 5,000.00/5,000 as zeros on the right side don't have any value!

Have A Nice Day!

Is this true? ....................

Answers

Answer:

no false.

Step-by-step explanation:

Find the value of h(-67) for the function below. h(x) = -49x − 125 A. 3,283 B. -3,408 C. 3,158 D. -1.18

Answers

Answer:

The answer is option C

Step-by-step explanation:

[tex]h(x) = - 49x - 125[/tex]

To find h(-67) , substitute the value of x that's - 67 into h(x). That is for every x in h(x) , replace it with - 67

We have

[tex]h( - 67) = - 49( - 67) + 125 \\ = 3283 - 125[/tex]

We have the final answer as

h( - 67) = 3158

Hope this helps you

The answer is option C

Six more than the product of seven and a number is 55.
Which equation and solution correctly represent this sentence?
7 x + 6 = 55; x = 7
7 X+6 = 55; X= 49
X/7 + 6 = 55: x= 7
X/7 + 6 = 55; X= 49

Answers


Six more than the product of seven and a number is 55.

The equation and solution correctly represent this sentence is :

7x + 6 = 55; x = 7.

Answer is the 2nd equation

The equation for the given situation is 7x+6=55 and the solution is x=7. Therefore, option A is the correct answer.

The given statement is six more than the product of seven and a number is 55.

What is an equation?

In mathematics, an equation is a formula that expresses the equality of two expressions, by connecting them with the equals sign =.

Let the unknown number be x.

Now, six more than the product of seven and a number is 55

7x+6=55

Solve the obtained equation.

That is, 7x=55-6

⇒7x=49

⇒x=7

The equation for the given situation is 7x+6=55 and the solution is x=7. Therefore, option A is the correct answer.

To learn more about an equation visit:

https://brainly.com/question/14686792.

#SPJ5

Alonzo works as a car detailer. He charges $165 per car. He is planning to move out of his parents' house and rent his first apartment. He will need to pay $105 for application fees, $895 for security deposit, and first and last months' rent at $1,120 per month. He has $1,919 in savings. How many cars must he detail to have enough money to rent the apartment?

Answers

Answer:

2 cars

Step-by-step explanation:

105 + 895 + 1120 = 2120

He has to pay 2120 of downpayment

2120 - 1919 = 201

Subtract what he already has

He needs 201 more dollars to rent his first apartment

He needs to do 1.21818182 cars...

You cant do 1/5 of a car so he has to do 2 cars

What is the solution to -7x-(-11-5x)=-4(2x-3)?
Thank you in advance!

Answers

Answer:

x=[tex]\frac{1}{6}[/tex]

Step-by-step explanation:

If you would like an explanation, let me know.

At Rhonda’s diner, 5 loaded baked potatoes and 4 cheeseburgers provide 4830 calories. 1 loaded baked potato and 4 cheeseburgers provide 2630 calories. Find the calorie content of each item.

Answers

Answer:

Potato: 550 calories

Cheeseburger: 520 calories

Step-by-step explanation:

Equation:

5p + 4c = 4830

p + 4c = 2630

5p + 4c = 4830

p + 4c = 2630  ----------> p = -4c + 2630

5(-4c+2630) + 4c = 4830

-20c + 13150 + 4c = 4830

-16c + 13150 = 4830

-16c = -8320

C = 520

Cheeseburger = 520 calories

p + 4(520) = 2630

p + 2080 = 2630

p = 550

Potato = 550 calories

convert 4/11 to a decimal using long division

Answers

Answer:

.36 repeating (both the 3&6)

Sorry my handwriting is bad!!

If 30 hundreds = tens and ones

Answers

the answer should be 10

Theo had 2 blocks. Then he got 2 more blocks from Ray and some more blocks from Lia.
Now Theo has 12 blocks.
The letter b stands for the number of blocks Theo got from Lla.
Which number sentence shows this word problem?
A. 2+b= 12
B. 2 + 12 = b
C. 2+2 + 12 = b
D. 2+2+b = 12

Answers

If I'm right it's (D). I hope this helps.

Ayudenme porfa!!!!!!!!!!!

Answers

Answer:

A. 2.5

B. 56 |\ 77

C. 15 /| 9

Which line segment is a reflection of AB in the x-axis?

Answers

Answer:

A) IJ

Step-by-step explanation:

IJ is the reflection

HELP PLEASE explain because i don’t understand why these are correct. (pic above) THANK YOU!!

Answers

Answer:

A, C, and E are correct answers because the other answers are decimals and you can't have a fraction of a car to wash.

does that make sense?

Hope this helps! Have a great day /night (wherever u r) : )

Bobby's measuring cup holds 27 in3 of flour. He has enough flour to fill this measuring cup exactly 120 times. If his bread recipe calls for 36 oz of flour per loaf, and we know that 1 in.3 of flour weighs 0.4 oz, how many loaves of bread can Bobby make?

Answers

Answer:

Bobby can make 36 loaves of bread.

Step-by-step explanation:

The measuring cup holds 27in³ of flour

Since he has enough flour to fill this measuring cup exactly 120 times,

Therefore, he has 3,240in³ (27in³ * 120) of flour.

Given that 1in³ of flour weighs 0.4 oz,

This implies that there are 1,296 oz (3,240in³ * 0.4 oz) of flour

And since the bread recipe per loaf consumes 36 oz of flour,

Therefore, the number of loaves Bobby can make is: 36 (1,296 oz / 36 oz)

Answer:

36 loaves of bread!

Other Questions
Is there anything in the study that lead you to believe that lacks Validity? explain your answer What does the word stalk mean on this sentence A group that we compare ourselves to for the purpose of evaluating our behaviors. The price of a sweater was reduced from $20 to $12. By what percentage was the price of the sweater reduced 18. When you turn off the power to a computer and unplug it at night, it loses the date, and you must reenter it each morning. What is the problem and how do you solve it? Two cars A and B with velocity 280m/s and 80m/s respectively, Car Bleaves point X 6:00am and car A also leaves the same point 6:40amcalculate the distance between the two cars after [I] 1hr [ii] where do youthink car A will overtake car B How does Atticus explain "courage" to Jem? How do YOU define "courage"? What is the value of f (-2) when f (x)=-2 (3 +x)? which element do you think is the most important element of your society ?why Starches are to simple sugars as proteins areA fatty acidsB) double bondsC) amino acidsD carbohydrates Describe the relationship between Squeaky and her brother Raymond. Because Buck has killed Spitz before the beginning of Chapters 4 and 5 his character can be said to have ________.completed his story arccompleted his character's development arc along a moral spectrumnever truly realized his potentialnot yet completed his story arc and will not achieve the full arc until the end of the story what are some good story ideas (for a short story) characters, plot, etc. HURRY I NEED HELP!!!worth 2 points)(04.07 HC)Listen, read the question, and choose the option with the correct answer.0:07 10:191xBased on the audio, qu es ms more barato?The tableThe rugThe napkinsThe pillowPrevious QuestionQuestion 1 (Answered)ONNext Question 4) 4) Determine the distance between -3 and 6.A) -9 units B) -3 units C) 3 units D) 9 units Effective presentations are organized into which three sections?opening, hook, closingopening, body, closingopening, outline, closing opening, body, checklist What were Aristotles 3 positives forms of government What was the purpose of the Populist movement?1- to fight the corruption of political machines in the cities2-to break rural isolation of farmers and represent their interests3-to halt the influx of immigrants from Southern and Eastern Europe4-to agitate for government ownership of railroads, telegraphs and telephones Plz help and explain why! I Will mark you brainliest if I can: If two angles are vertical angles, then they are... a.) complementaryb.) supplementaryc.) congruentd.) perpendiculare.) parallelf.) corresponding anglesI put corresponding angles and it was wrong. Complete the table by listing at least one of the questions associated with each step in an artcritique. (APEX) PLEASE I NEED HELP WITH THIS ASAP I WILL EVEN SEND YOU MONEY(through PayPal)